During a math class, a proposed problem on a handout is as follows:
Prove that \[\lfloor x\rfloor +\dfrac{\lfloor 2x\rfloor}{2}+\dfrac{\lfloor 3x\rfloor}{3}+\cdots +\dfrac{\lfloor nx\rfloor}{n}\le \lfloor nx\rfloor\] for all positive reals \(x\) and integer \(n\).
Can someone help me prove it? I tried to use a periodic argument, proving it true for then plugging in for positive integer to prove the inequality for all other . However it didn't work, because my inequality substitutions were too strict and I ended up with which is not true.
Thanks!
Easy Math Editor
This discussion board is a place to discuss our Daily Challenges and the math and science related to those challenges. Explanations are more than just a solution — they should explain the steps and thinking strategies that you used to obtain the solution. Comments should further the discussion of math and science.
When posting on Brilliant:
*italics*
or_italics_
**bold**
or__bold__
paragraph 1
paragraph 2
[example link](https://brilliant.org)
> This is a quote
\(
...\)
or\[
...\]
to ensure proper formatting.2 \times 3
2^{34}
a_{i-1}
\frac{2}{3}
\sqrt{2}
\sum_{i=1}^3
\sin \theta
\boxed{123}
Comments
http://mks.mff.cuni.cz/kalva/usa/usoln/usol815.html
Apparently this is USAMO 1981 #5.
Log in to reply
Yep.
Log in to reply
I was just gonna say...
I think using induction approach should also work
I don't know if I am right or wrong ... So please correct me ... Thanks in advance ... here is my Solution : As we know, floor(x) = x - {x} , where {x} is fractional part. if we replace all the floor values in the above eqn will reduce to
LHS : nx - [ {x} + ({2x}/2) + ({3x}/3) + ({4x}/4) + ........... + ({nx}/n) ]
What is interesting to see is that {x} = {2x}/2 = {3x}/3 = ..... = {nx}/n (You can check it with any value of x :) )
Hence LHS will become : nx - n{x}
Solving RHS we got : floor(nx) = nx - {nx} As the fractional part of {nx} will always lie between 0 and 1, i.e. 0 <= {nx} < 1
And it is easy to see that the product of n and {x} in LHS is always greater than or equal to (in case of 0 as fractional value) {nx} in RHS.
Probably this will prove it ... What say guys ???
Log in to reply
@Daniel Liu @Finn Hulse : Guys can you please check the above solution ???
Log in to reply
I still don't think it proves it. @Finn Hulse
Your solution depends on the fact that {x}+2{2x}+⋯+n{nx}≥{nx} but you didn't prove that.
Log in to reply
It's good as far as I can tell! :D
n{x}={nx}. Try, e.g., x=0.6, n=2. Then 2⋅{0.6}=1.2={2⋅0.6}=0.2.
Log in to reply
When did I say n{x} = {nx} ??? Please check it once more :)
Log in to reply
I could have equally said that {x}=n{nx}, just multiply both sides of your equality by n (we can, since n=0). Both of the equalities are equivalent.
Log in to reply
that should prove it man :)...... i don't find any serious anomaly..... good job
Log in to reply
{x} = {2x}/2 = {3x}/3 = ..... = {nx}/n is false (see my other comments).
Log in to reply